Вы находитесь на странице: 1из 2

c Christophe Bertault - MPSI Ensembles finis et dnombrement

Exercice 1
On tire simultanment 6 cartes dun jeu de tarot 21 atouts, la carte quon appelle
l excuse , et 14 cartes par couleur, sachant quil y a 4 couleurs, comme toujours. On
ne cherchera pas valuer numriquement les rsultats obtenus dans cet exercice, sauf si
on aime se compliquer la vie. Combien de tirages dirents peut-on obtenir contenant :
1) deux atouts et quatre tres ?
2) six carreaux, ou bien trois carreaux, deux piques et lexcuse ?
3) exactement un atout et au moins trois as ?
4) au plus un cur et au moins quatre atouts ?
Exercice 2
On appellera mot toute suite de lettres, quelle ait un sens ou non. On rappelle que
la lettre y est une voyelle.
1) Combien de mots de 5 lettres peut-on former avec les 26 lettres de lalphabet, dans
lesquels toute consonne est suivie dune voyelle et toute voyelle dune consonne ?
2) Dterminer le cardinal de lensemble E des mots de 5 lettres forms partir des
26 lettres de lalphabet et tels que :
(i) la lettre q est forcment suivie de la lettre u ,
(ii) toute consonne est suivie dune voyelle,
(iii) toute voyelle est suivie dune consonne, une exception prs : les u
qui sont conscutifs dun q sont suivis dune voyelle autre que u ,
(iv) les deux dernires lettres ne peuvent pas tre qu .
Exercice 3
Soient A1, A2, . . . , An des ensembles. Prouver la formule du crible (ou formule de Poin-
car) :

n
_
i=1
Ai

=
n

k=1
(1)
k+1

1i
1
<i
2
<...<i
k
n

Ai
1
Ai
2
. . . Ai
k

.
Quel formule retrouve-t-on dans le cas o les Ai, i 1, n, sont deux deux disjoints ?
Exercice 4
Soient n, p N

tels que p n. Montrer, par un raisonnement combinatoire, que :


_
n
p
_
=
_
n
n p
_
et
_
n
p 1
_
+
_
n
p
_
=
_
n + 1
p
_
.
Exercice 5
Soient n, p N

. Combien existe-t-il dapplications strictement croissantes de 1, n dans


1, p ?
Exercice 6
1) Soient E un ensemble ni de cardinal n et p, k N tels que 0 k p n.
Calculer de deux faons le nombre de couples (A, B) P(E)
2
tels que A B,
|A| = k et |B| = p.
En dduire que
_
n
k
__
n k
p k
_
=
_
n
p
__
p
k
_
, puis que
_
n
k
__
n k
n p
_
=
_
n
p
__
p
k
_
.
2) On note S lapplication de R
N
dans lui-mme qui, toute suite relle (xn)
nN
,
associe la suite (yn)
nN
dnie par : n N, yn =
n

k=0
_
n
k
_
x
k
et T lap-
plication de R
N
dans lui-mme qui, toute suite relle (x

n
)
nN
, associe la suite
(y

n
)
nN
dnie par : n N, y

n
=
n

k=0
(1)
nk
_
n
k
_
x

k
.
Montrer que S et T sont deux bijections de R
N
dans R
N
inverses lune de lautre.
Exercice 7
1) On considre un ensemble ni E dont les lments sont de deux types nots 1 et
2. Prcisment, E contient n1 lments de type 1 et n2 lments de type 2. Soit
k 0, n1 +n2. Exprimer de deux faons le nombre de parties k lments de E
que lon peut former. En dduire une jolie une formule.
2) Simplier
n

k=0
_
n
k
_
2
pour tout n N.
Exercice 8
Soient n, p N. Pour tout k 1, n+1, dterminer le nombre de parties de 1, n+p+1
de cardinal p + 1 et de maximum p +k. Simplier ainsi
n

k=0
_
p +k
p
_
et
n

k=0
_
p +k
k
_
.
Exercice 9
1) Soient G un groupe ni et x G.
a) Montrer que x
n
= 1G pour un certain n N

.
b) On appelle ordre de x et on note o(x) le plus petit lment de lensemble
_
n N

/ x
n
= 1G
_
. Justier lexistence de o(x).
c) Montrer que lensemble
_
1G, x, x
2
, . . . , x
o(x)1
_
est un sous-groupe de G. On
note
_
x
_
ce sous-groupe et on lappelle le sous-groupe de G engendr par x.
2) Soit E un magma associatif ni. Montrer que E possde un idempotent, i.e. un
lment x E pour lequel x
2
= x.
Exercice 10
Soit E un ensemble. On associe, toute partie A de E, lapplication lindicatrice de A
A : E
_
0, 1
_
dnie par : x E, A(x) =
_
1 si x A
0 si x / A.
Montrer que lapplication A A est bijective de P(E) sur
_
0, 1
_
E
. Retrouver ainsi,
dans le cas o E est ni, un rsultat du cours.
1
c Christophe Bertault - MPSI Ensembles finis et dnombrement
Exercice 11
1) Soit n N

. On range n +1 chaussettes dans n tiroirs. Montrer quau moins un


tiroir contient deux chaussettes. Ce rsultat simple et essentiel est connu sous le
nom de principe des tiroirs.
2) a) Calculer

3


4
puis tan

12
.
b) Montrer qutant donns 13 rels, on peut toujours en trouver deux x et y
pour lesquels 0 <
x y
1 +xy
< 2

3.
3) a) Soient x R et n N

. Pour tout k 0, n, on pose


k
= kxkx, lment
de [0, 1[. Appliquer le principe des tiroirs aux rels
k
, k dcrivant 0, n, pour
montrer le thorme dapproximation de Dirichlet suivant :
(p, q) Z N

/ q n et

x
p
q

<
1
nq
.
b) En dduire que pour tout x R, lensemble des couples (p, q) Z N

pour
lesquels

x
p
q

<
1
q
2
est inni.
c) Dmontrer le thorme de Bzout partir de la question a) : pour tous a, b Z
premiers entre eux, il existe u, v Z tels que au +bv = 1.
d) On admet lirrationnalit de , de sorte que sin n = 0 pour tout n N

. La
suite
_
1
nsin n
_
nN

possde-t-elle une limite, et si oui laquelle ?


Exercice 12
Soient p et q deux irrationnels strictement positifs tels que
1
p
+
1
q
= 1. On souhaite
prouver le thorme de Beatty selon lequel les ensembles
_
np
_
nN

et
_
nq
_
nN

forment une partition de N

, i.e. sont disjoints de runion N

tout entier.
On pose P =
_
np
_
nN

et Q =
_
nq
_
nN

.
1) Montrer que les ensembles P et Q sont disjoints.
2) En dduire que (P Q) [1, N] est de cardinal N 1 pour tout N N

.
3) Conclure.
Exercice 13
Soit n N

. On pose G =
_
Sn/ k 1, n + 1, (n k + 1) = n (k) + 1
_
.
1) Montrer que G est un sous-groupe de Sn.
2) Calculer le cardinal de G.
Exercice 14
On se donne n entiers relatifs quelconques. Montrer que lon peut toujours former par
somme, en choisissant certains dentre eux, un multiple de n.
2

Вам также может понравиться